Difference between revisions of "2011 AMC 10B Problems/Problem 15"
(→Solution) |
Erics son07 (talk | contribs) (→Solution) |
||
Line 27: | Line 27: | ||
<math>\boxed{\textbf{(E)} \text{II and III only}}</math> | <math>\boxed{\textbf{(E)} \text{II and III only}}</math> | ||
+ | |||
+ | ==Solution 2== | ||
+ | Alternatively, substitute arbitrary values for <math>x</math>, <math>y</math>, and <math>z</math> where <math>x\neq y\neq z</math>. For example, <math>x=1</math>, <math>y=2</math>, and <math>z=3</math> give that only II and III work. <math>\boxed{\textbf{(E)} \text{II and III only}}</math> | ||
== See Also== | == See Also== |
Latest revision as of 00:48, 26 June 2022
Contents
Problem
Let denote the "averaged with" operation: . Which of the following distributive laws hold for all numbers and ?
Solution
Simplify each operation and see which ones hold true.
Solution 2
Alternatively, substitute arbitrary values for , , and where . For example, , , and give that only II and III work.
See Also
2011 AMC 10B (Problems • Answer Key • Resources) | ||
Preceded by Problem 14 |
Followed by Problem 16 | |
1 • 2 • 3 • 4 • 5 • 6 • 7 • 8 • 9 • 10 • 11 • 12 • 13 • 14 • 15 • 16 • 17 • 18 • 19 • 20 • 21 • 22 • 23 • 24 • 25 | ||
All AMC 10 Problems and Solutions |
The problems on this page are copyrighted by the Mathematical Association of America's American Mathematics Competitions.